Difference between revisions of "2011 AMC 8 Problems/Problem 4"
m |
|||
Line 1: | Line 1: | ||
Here is a list of the numbers of fish that Tyler caught in nine outings last summer: <cmath>2,0,1,3,0,3,3,1,2.</cmath> Which statement about the mean, median, and mode is true? | Here is a list of the numbers of fish that Tyler caught in nine outings last summer: <cmath>2,0,1,3,0,3,3,1,2.</cmath> Which statement about the mean, median, and mode is true? | ||
− | <math>\ | + | <math>\textbf{(A)}\ \text{median} < \text{mean} < \text{mode} \qquad \textbf{(B)}\ \text{mean} < \text{mode} < \text{median} \\ \\ \textbf{(C)}\ \text{mean} < \text{median} < \text{mode} \qquad \textbf{(D)}\ \text{median} < \text{mode} < \text{mean} \\ \\ \textbf{(E)}\ \text{mode} < \text{median} < \text{mean}</math> |
==Solution== | ==Solution== | ||
+ | |||
+ | First, put the numbers in increasing order. | ||
+ | |||
+ | <cmath>0,0,1,1,2,2,3,3,3</cmath> | ||
+ | |||
+ | The mean is <math>\frac{0+0+1+1+2+2+3+3+3}{9} = \frac{15}{9},</math> the median is <math>2,</math> and the mode is <math>3.</math> Because, <math>\frac{15}{9} < 2 < 3,</math> the answer is <math>\boxed{\textbf{(C)}\ \text{mean} < \text{median} < \text{mode}}</math> | ||
==See Also== | ==See Also== | ||
{{AMC8 box|year=2011|num-b=3|num-a=5}} | {{AMC8 box|year=2011|num-b=3|num-a=5}} |
Revision as of 17:36, 25 November 2011
Here is a list of the numbers of fish that Tyler caught in nine outings last summer: Which statement about the mean, median, and mode is true?
Solution
First, put the numbers in increasing order.
The mean is the median is and the mode is Because, the answer is
See Also
2011 AMC 8 (Problems • Answer Key • Resources) | ||
Preceded by Problem 3 |
Followed by Problem 5 | |
1 • 2 • 3 • 4 • 5 • 6 • 7 • 8 • 9 • 10 • 11 • 12 • 13 • 14 • 15 • 16 • 17 • 18 • 19 • 20 • 21 • 22 • 23 • 24 • 25 | ||
All AJHSME/AMC 8 Problems and Solutions |